0% found this document useful (0 votes)
1K views46 pages

Mam 5P2Z6Uivs9MldDbD4w

The document describes the problems and solutions from the 25th Junior Balkan Mathematical Olympiad. It provides information about the contributing countries, problem selection committee members, and 7 algebra, 5 geometry, and 3 number theory problems proposed for the competition.

Uploaded by

mathlinks1990
Copyright
© © All Rights Reserved
We take content rights seriously. If you suspect this is your content, claim it here.
Available Formats
Download as PDF, TXT or read online on Scribd
0% found this document useful (0 votes)
1K views46 pages

Mam 5P2Z6Uivs9MldDbD4w

The document describes the problems and solutions from the 25th Junior Balkan Mathematical Olympiad. It provides information about the contributing countries, problem selection committee members, and 7 algebra, 5 geometry, and 3 number theory problems proposed for the competition.

Uploaded by

mathlinks1990
Copyright
© © All Rights Reserved
We take content rights seriously. If you suspect this is your content, claim it here.
Available Formats
Download as PDF, TXT or read online on Scribd
You are on page 1/ 46

The 25th JUNIOR BALKAN MATHEMATICAL OLYMPIAD

Chișinău, Republic of Moldova


June 29 – July 5, 2021

SHORTLIST
OF SELECTED PROBLEMS
AND SOLUTIONS
The 25th Junior Balkan Mathematical Olympiad (JBMO 2021)

NOTE OF CONFIDENTIALITY

The shortlisted problems should be kept


strictly confidential until JBMO 2022.

CONTRIBUTING COUNTRIES
The Organizing Committee and the Problem Selection Committee of the
JBMO 2021 thank the following 7 countries for contributing 22 problem
proposals:
Azerbaijan
Bosnia and Herzegovina
Bulgaria
Cyprus
France
North Macedonia
Turkey

PROBLEM SELECTION COMMITTEE


Chairperson: Baltag Valeriu
Members: Costaș Ana
Croitoru Dorian
Cuznețov Eugeniu
Guțu Valeriu
Lopotenco Alexandru
Rusu Galina
JBMO 2021 Problems 3

ALGEBRA

A1. Consider the equation


¹ º
1
2 − n + 1 = (n + 1)(1 − nx),
2x

where n is a positive integer and x is the unknown nonzero real variable.


a) Solve the equation when: i) n = 8; ii) n = 51.
b) Prove that for some integer N and all integers n ≥ N the equation has at least
2021 solutions.
(For a real number y with byc we denote the largest integer K such that K ≤ y.)
(Proposed by Bulgaria)

A1B1 . Let n (n ≥ 1) be an integer. Consider the equation


¹ º
1
2· − n + 1 = (n + 1)(1 − nx),
2x

where x is the unknown real variable.


a) Solve the equation for n = 8.
b) Prove that there exists an integer k (k ≥ 1) such that for all integers n ≥ k the
equation has at least 2021 solutions.
(For any real number y by byc we denote the largest integer m such that m ≤ y.)

A2. Let n > 3 be a positive integer. Find all integers k such that 1 6 k 6 n and for
which the following property holds:

If x1 , . . . , xn are n real numbers such that xi + xi+1 + . . . + xi+k−1 = 0 for all


integers i > 1 (indexes are taken modulo n), then x1 = . . . = xn = 0.

(Proposed by Vincent Jugé and Théo Lenoir, France)

A2B2 . Let n > 3 be an integer. Find all integers k (1 6 k 6 n) for which the following
property holds:

If n real numbers are placed around the circle such that the sum of any k
consecutively placed of these numbers equals 0, then all these numbers are
necessarily 0.

1
Proposed by PSC (Problem Selection Committee).
2
Proposed by PSC.
JBMO 2021 Problems 4

A3. Let n be a positive integer. A finite set of integers is called n-divided if there is
exactly n ways to partition this set into two subsets with equal sums.
For example, the set {1, 3, 4, 5, 6, 7} is 2-divided because the only ways to partition it
into two subsets with equal sums is by dividing it into {1, 3, 4, 5} and {6, 7}, or {1, 5, 7}
and {3, 4, 6}.
Find all the integers n > 0 for which there exists a n-divided set.
(Proposed by Martin Rakovsky, France)

A3B3 . Let n (n ≥ 1) be an integer. A finite set of integers is called n-divided if there


are exactly n different partitions of this set into two subsets with equal sums of their
elements.
For example, the set {1, 3, 4, 5, 6, 7} is 2-divided because the are only two its partitions
into two subsets with equal sums, namely the first is: {1, 3, 4, 5} and {6, 7}, the second
is: {1, 5, 7} and {3, 4, 6}.
Find all the integers n ≥ 1 for which there exists a n-divided set.
(Two partitions of the set are considered to be different if at least one subset of one
partition does not coincide with any subset of another partition.)

3
Proposed by PSC.
JBMO 2021 Problems 5

GEOMETRY

G1. Let ABC be an acute scalene triangle with circumcenter O. Let D be foot of an
A-altitude in triangle ABC and let E denote intersection of lines BC and AO. Let l be
a line through E perpendicular to AO. Let l intersect AB and AC at K, L respectively.
Denote by ω circumcircle of triangle AKL. Line AD intersects ω again at X. Prove that
circumcles of triangles ABC, DEX and ω have a common point.
(Proposed by Boris Stanković, Bosnia and Herzegovina)

G1B.4 Let ABC be an acute scalene triangle with circumcenter O. Let D be the foot
of the altitude from A to the side BC. The lines BC and AO intersect at E. Let l be
the line through E perpendicular to AO. The line l intersects AB and AC at K and L,
respectively. Denote by ω the circumcircle of triangle AKL. Line AD intersects ω again
at X. Prove that ω and the circumcircles of triangles ABC and DEX have a common
point.

b = 90◦ . If
G2. Let P be an interior point of the isosceles triangle ABC with A

P[
AB + \
P BC + P[
CA = 90◦ ,

prove that AP ⊥ BC.


(Proposed by Mehmet Akif Yıldız, Turkey)

G2B5 . Let P be a point inside the isosceles triangle ABC with ∠BAC = 90◦ . Prove that
the lines AP and BC are perpendicular if and only if

∠P AB + ∠P BC + ∠P CA = 90◦ .

G3. Let ABC be an acute triangle with circumcircle ω and circumcenter O. The perpen-
dicular from A to BC intersects BC and ω at D and E, respectively. Let F be a point
on the segment AE, such that 2 · F D = AE. Let l be the perpendicular to OF through
F . Prove that l, the tangent to ω at E and the line BC are concurrent.
(Proposed by Stefan Lozanovski, North Macedonia)

G3B6 . Let ABC be an acute triangle with circumcircle ω and circumcenter O. The
perpendicular from A to BC intersects BC at D, and it intersects ω again at E. Let F
be a point on the segment AE, such that AE = 2 · F D. Let l be the perpendicular to OF
through F . Prove that l, the line BC and the tangent to ω at E have a common point.

4
Proposed by PSC.
5
Proposed by PSC.
6
Proposed by PSC.
JBMO 2021 Problems 6

G4. Let ABCD be a convex quadrilateral with ∠B = ∠D = 90◦ . Let E be the point
of intersection of BC with AD and let M be the midppoint of AE. On the extension of
CD, beyond the point D, we pick a pint Z such that M Z = AE 2
. Let U and V be the
projections of A and E respectively on BZ. The circumcircle of the triangle DU V meets
again AE at the point L. If I is the point of intersection of BZ with AE, prove that the
lines BL and CI intersect on the line AZ.
(Proposed by Cyprus)

G4B7 . Let ABCD be a convex quadrilateral with ∠ABC = ∠ADC = 90◦ . The perpen-
diculars from A and C to BD intersect BD at U and V , respectively. The perpendicular
from D to AC intersects AC and BC at T and X, respectively. The circumcircle of
triangle T U V intersects AC again at R. The lines AC and BD intersect at F . Prove
that the lines XF , BR and AD have a common point.

G5. Let ABC be an acute scalene triangle with circumcircle ω. A line parallel to BC
cuts AB and AC in P and Q respectively. Let L be a point on ω such that AL k BC.
Denote by S the intersection of segments BQ and CP . If K is the intersection of LS
with ω, prove that ∠BKP = ∠CKQ.
(Proposed by Ervin Macić, Bosnia and Herzegovina)

G5B8 . Let ABC be an acute scalene triangle with circumcircle ω. Let P and Q be interior
points of the sides AB and AC, respectively, such that P Q is parallel to BC. Let L be
a point on ω such that AL is parallel to BC. The segments BQ and CP intersect at S.
The line LS intersects ω at K. Prove that ∠BKP = ∠CKQ.

7
Proposed by PSC.
8
Proposed by PSC.
JBMO 2021 Problems 7

NUMBER THEORY

NT1. Find all positive integers a, b, c such that ab + 1, bc + 1 and ca + 1 are all equal to
factorials of some positive integers.
(Proposed by Nikola Velov, North Macedonia)

NT2. The real numbers x, y and z are such that x2 + y 2 + z 2 = 1.


a) Determine the smallest and the largest possible values of xy + yz − xz.
b) Prove that there does not exist a triple (x, y, z) of rational numbers, which attains
any of the two values in a).
(Proposed by Bulgaria)

NT2B9 . The real numbers x, y and z are such that x2 + y 2 + z 2 = 1. Let s and L be the
smallest and the largest possible values of xy + yz − xz.
a) Determine s and L.
b) Prove that there does not exist a triple (x, y, z) of rational numbers, which attains
any of the two values s and L.

NT3. For given set A = {x1 , x2 , x3 , x4 , x5 } of five distinct positive integers denote sum
of its elements with SA . Let TA denote number of triplets (i, j, k) with 1 6 i < j < k 6 5
for which xi + xj + xk divides SA . Among all sets of five distinct positive integers find,
with proof, maximum value that TA can attain.
(Proposed by Boris Stanković, Bosnia and Herzegovina)

NT3B10 . For any set A = {x1 , x2 , x3 , x4 , x5 } of five distinct positive integers denote
by SA the sum of its elements, and denote by TA the number of triplets (i, j, k) with
1 6 i < j < k 6 5 for which xi + xj + xk divides SA . Find the largest possible value of TA .

9
Proposed by PSC.
10
Proposed by PSC.
JBMO 2021 Problems 8

NT4. Dragoş, the early ruler of Moldavia, and Maria the Oracle play the following
game. Firstly, Maria chooses a set S of prime numbers. Then Dragoş gives an infinite
sequence x1 , x2 , . . . of distinct positive integers. Then Maria picks a positive integer M
and a prime number p from her set S. Finally, Dragoş picks a positive integer N and the
game ends. Dragoş wins if and only if for all integers n ≥ N the number xn is divisible
by pM ; otherwise, Maria wins.
Who has a winning strategy if the set S must be: a) finite; b) infinite?
(Proposed by Bulgaria)

NT4B11 . Dragoş and Maria play the following game. Firstly, Maria chooses a nonempty
set S of prime numbers. Dragoş, knowing the chosen set S, gives an infinite sequence
x1 , x2 , . . . of distinct positive integers.
Dragoş wins if and only if for any positive integer m and any prime number p ∈ S,
there exists a positive integer k such that for any integer n ≥ k the number xn is divisible
by pm ; otherwise, Maria wins.
Who has a winning strategy if:
a) the set S is finite; b) the set S is infinite?

NT5. Find all pairs of integers (x, y) such that x2 + 5y 2 = 2021y.


(Proposed by Bulgaria)

NT6. Given a positive integer n > 2, we define f (n) to be the sum of all remainders
obtained by dividing n by all positive integers less than n. For example dividing 5 with
1, 2, 3 and 4 we have remainders equal to 0, 1, 2 and 1 respectively. Therefore f (5) =
0 + 1 + 2 + 1 = 4.
Find all positive integers n > 3 such that f (n) = f (n − 1) + (n − 2).
(Proposed by Cyprus)

NT6B12 . Given a positive integer n > 2, we denote by fn the sum of all remainders
obtained by dividing n by all positive integers less than n. For example, dividing 5
with 1, 2, 3 and 4, we have remainders equal to 0, 1, 2 and 1 respectively. Therefore
f5 = 0 + 1 + 2 + 1 = 4.
Find all positive integers n > 2 such that fn+1 − fn = n − 1.

11
Proposed by PSC.
12
Proposed by PSC.
JBMO 2021 Problems 9

NT7. Alice chooses a prime number p > 2 and then Bob chooses a positive integer
n0 . Alice, in the first move, chooses an integer n1 > n0 and calculates the expression
s1 = nn0 1 + nn1 0 ; then Bob, in the second move, chooses an integer n2 > n1 and calculates
the expression s2 = nn1 2 + nn2 1 ; etc. one by one. (Each player knows the numbers chosen
by the other in the previous moves.) The winner is the one who first chooses the number
nk such that p divides sk (s1 + 2s2 + · · · + ksk ). Who has a winning strategy?
(Proposed by Borche Joshevski, North Macedonia)

NT7B13 . Alice and Bob play a game. At her initial move, Alice chooses a prime number
p > 2 and then Bob chooses a positive integer n0 . After that, Alice, in her standard move,
chooses an integer n1 > n0 and calculates the expression s1 = nn0 1 + nn1 0 . Then Bob, in his
standard move, chooses an integer n2 > n1 and calculates the expression s2 = nn1 2 + nn2 1 .
They repeat one by one their standard moves. Each player knows the numbers chosen by
the other in the previous moves. The winner is the player who first chooses the number
nk such that p divides sk (s1 + 2s2 + · · · + ksk ). Who has a winning strategy?

13
Proposed by PSC.
JBMO 2021 Problems 10

COMBINATORICS

C1+NT. In Mathcity, there are infinitely many buses and infinitely many stations. The
stations are indexed by the powers of 2: 1, 2, 4, 8, 16, . . . Each bus goes by finitely many
stations, and the bus number is the sum of all the stations it goes by. For simplifications,
the mayor of Mathcity wishes that the bus numbers form an arithmetic progression with
common difference r and whose first term is the favourite number of the mayor.
For which positive integers r is it always possible that, no matter the favourite number
of the mayor, given any m stations, there is a bus going by all of them?
(Proposed by Savinien Kreczman and Martin Rakovsky, France)

C1B+NT14 . In Mathcity, there are infinitely many buses and infinitely many stations.
All stations are indexed by the different powers of 2: S = {1, 2, 4, 8, 16, . . .}, such that
each element of S is the index for some station. Each bus stops by finitely many distinct
stations, and the bus number is the sum of indices of all the stations it stops by. The bus
numbers form an arithmetic progression with common difference r and whose first term
is the favorite number of the mayor.
Find all positive integers r such that for any finite set of stations, no matter the favorite
number of the mayor, there is always a bus stopping by these stations (and, possibly, other
stations)?

C2. Let n be a positive integer. We are given a 3n × 3n board whose unit squares are
colored in black and white in such way that starting with the top left square, every third
diagonal is colored in black and the rest of the board is in white. In one move, one can
take a 2 × 2 square and change the color of all its squares in such way that white squares
become orange, orange ones become black and black ones become white. Find all n for
which, using a finite number of moves, we can make all the squares which were initially
black white, and all squares which were initially white black.

(Proposed by Boris Stanković and Marko Dimitrić, Bosnia and Herzegovina)

14
Proposed by PSC.
JBMO 2021 Problems 11

C3. We have a set of 343 closed jars, each containing blue, yellow and red marbles with
the number of marbles from each color being at least 1 and at most 7. No two jars have
exactly the same contents. Initially all jars are with the caps up. To flip a jar will mean
to change its position from cap-up to cap-down or vice versa. It is allowed to choose a
triple of positive integers (b; y; r) ∈ {1; 2; ...; 7}3 and flip all the jars whose number of blue,
yellow and red marbles differ by not more than 1 from b, y, r, respectively. After n moves
all the jars turned out to be with the caps down. Find the number of all possible values
of n, if n ≤ 2021.
(Proposed by Bulgaria)

C3B15 . There are 343 closed jars, each containing blue, yellow and red marbles with the
number of marbles from each color being at least 1 and at most 7. No two jars have
exactly the same contents. Initially all jars are with the caps up. To flip a jar will mean
to change its position from cap-up to cap-down or vice versa. It is allowed to choose a
triple of positive integers (b, y, r) with b, y, r ∈ {1, 2, . . . , 7} and flip all jars whose numbers
of blue, yellow and red marbles differ by not more than 1 from b, y and r, respectively.
After n moves all the jars turned out to be with the caps down.
a) Find the smallest possible value of n.
b) Is it possible that n = 2021 ?

C4. Alice and Bob play a game together as a team on a 100 × 100 board with all unit
squares initially white. Alice sets up the game by coloring exactly k of the unit squares
red at the beginning. After that, a legal move for Bob is to choose a row or column
with at least 10 red squares and color all of the remaining squares in it red. What is the
smallest k such that Alice can set up a game in such a way that Bob can color the entire
board red after finitely many moves.
(Proposed by Nikola Velov, North Macedonia)

C4B16 . Alice plays a game on a 100 × 100 board with all unit squares 1 × 1 initially white.
First, Alice chooses k unit squares and colors them in red. After, at each her move, Alice
chooses a row or a column with at least 10 red squares (if there is) and colors in red all
of the remaining white squares (if there are) in this row or column. It is known that after
finitely many moves Alice succeeded to color in red all the unit squares of the table. Find
the smallest possible value of k.

C5. Let A be a subset of the set of 2021 integers {1, 2, 3, . . . , 2021} such that whenever
a, b, c are three not necessarily distinct elements of A, then |a + b − c| > 10.
What is the largest possible number of elements of A?
(Proposed by Cyprus)
15
Proposed by PSC.
16
Proposed by PSC.
JBMO 2021 Problems 12

C6. Given an m × n table consisting of mn unit cells. Alice and Bob play the following
game: Alice goes first and the one who makes move colors one of the empty cells with

one of the given three colors. Alice wins if there is a or figure having three
different colors. Otherwise Bob is the winner. Determine the winner for all cases of m
and n where m, n ≥ 3.
(Proposed by Toghrul Abbasov, Azerbaijan)
JBMO 2021 Algebra. Solutions 13

SOLUTIONS

ALGEBRA. SOLUTIONS
A1. Let n be a positive integer. Consider the equation
¹ º
1
2 − n + 1 = (n + 1)(1 − nx),
2x
where x is the nonzero real variable.
a) Solve the equation when: i) n = 8; ii) n = 51.
b) Prove that for some integer N and all integers n ≥ N the equation has at least
2021 solutions.
(For a real number y, byc denotes the largest integer K such that K ≤ y.)
(Proposed by Bulgaria)
17
Remark. Feel free to remove/add suitable parts – e.g. if in your opinion b) becomes
too obvious after having solved a) ii), you could remove a) ii).
Solution. Let A = b 2x 1
c – then the equation gives x = 2(n−A)
n(n+1)
and now substituting in
the definition of A yields ¹ º
n(n + 1)
A= .
4(n − A)
The latter equality is a necessary and sufficient condition for the corresponding x to be a
solution to the equation. Let us also observe that A is an integer and that 1 ≤ A ≤ n − 1
for n ≥ 3 – indeed, if A = 0, then 0 = b n+1 4
c ≥ 1; if A = n, the right-hand side is
undefined; and if A < 0 or A > n, then the sides have different signs.
18
a) i) For n = 8 we want A = b 8−A c. By the above, A is an integer between 1 and 7
inclusive. A direct verification shows that only A = 3 and A = 4 are solutions, with the
5
corresponding x being x = 36 and x = 19 .
663
ii) For n = 51 we want A = b 51−A c for integers 1 ≤ A ≤ 50. This holds if and only if
663
A ≤ 51−A < A + 1. The left inequality is equivalent to (2A − 51)2 + 51 ≥ 0 and holds for
all A. The right one is equivalent to (A − 25)2 < 13 and hence has only 22 ≤ A ≤ 28 as
solutions. Hence all solutions are x = 51−A
26·51
for 22 ≤ A ≤ 28.
n(n+1)
b) It suffices to have at least 2021 integer solutions 1 ≤ A ≤ n−1 to A ≤ 4(n−A) < A+1
2
whenever n ≥ N for some suitable N . The left inequality is equivalent to (2A−n) +n ≥ 0
and holds for all A. The√right inequality is√equivalent to (2A − n + 1)2 < n + 1 and hence
holds precisely for n−1−2 n+1 < A < n−1+2 n+1 . Observe that this range for A is tighter
than 1 ≤ A ≤ n − 1 for n ≥ 6, as (n − 3)2 > n + 1 and (n − 1)2 >√(n + 1) for √
these
n−1− n+1 n−1+ n+1
n. Finally, the difference between the endpoints of the interval ( , ) is
√ 2 2
n + 1 and hence for sufficiently large n this interval must contain at least 2021 integers.
This completes the proof. ¤
17
All remarks are made by the problem authors.
JBMO 2021 Algebra. Solutions 14

A2. Let n > 3 be a positive integer. Find all integers k such that 1 6 k 6 n and for
which the following property holds:

If x1 , . . . , xn are n real numbers such that xi + xi+1 + . . . + xi+k−1 = 0 for all


integers i > 1 (indexes are taken modulo n), then x1 = . . . = xn = 0.

(Proposed by Martin Rakovsky, France)

Answer: All integers k such that 1 6 k 6 n and k is coprime with n.

Solution. First, if some integer d > 2 divides both k and n, the sequence

x1 , x2 , . . . , xn = 1, 0, . . . , 0, −1, 1, 0, . . . , 0, −1, . . . , 1, 0, . . . , 0, −1
| {z } | {z } | {z }
d numbers d numbers d numbers

is such that xi + xi+1 + . . . + xi+k−1 = 0 for all integers i > 1, but it contains non-zero
terms. Thus, if k is not coprime to n, it cannot be a solution of the problem.
Now, consider some integer k ∈ {1, 2, . . . , n} that is coprime with n, and let x1 , x2 , . . . , xn
be real numbers such that xi + xi+1 + . . . + xi+k−1 = 0 for all integers i > 1. Given any
integer i, we have
xi = −xi+1 − . . . − xi+k−1 = xi+k .
Thus, the sequence (xm )m>1 is periodic, with period k. Since k is coprime with n,
there exists an integer ` such that k` ≡ 1 (mod n). It follows that xi = xi+k` = xi+1 for
all i > 1, i.e., that the real numbers xi are all equal. Hence, 0 = x1 + x2 + . . . + xk = kx1
and 0 = x1 = x2 = . . . = xn .
In conclusion, the solutions are the integers k such that 1 6 k 6 n and k is coprime
with n. ¤
JBMO 2021 Algebra. Solutions 15

A3. Let n be a nonnegative integer. A finite set of integers is called n-divided if there
are exactly n ways to partition this set into two subsets with equal sums.
For example, the set {1, 3, 4, 5, 6, 7} is 2-divided because the only ways to partition it
into two subsets with equal sums is by dividing it into {1, 3, 4, 5} and {6, 7}, or {1, 5, 7}
and {3, 4, 6}.
Find all the integers n > 0 for which there exists a n-divided set.
(Proposed by Vincent Jugé and Théo Lenoir, France)

Answer: All integers n > 0.

Solution. First, note that the set {1} is 0-divided and the set {1, 2, 3} is 1-divided.
Now consider an integer n > 2 and let us show that the set

E = {k : 3n 6 k 6 4n − 1} ∪ {k : 4n + 1 6 k 6 5n} ∪ {8n(n − 2)}

is n-divided. Indeed, if ΣX denotes the sum of the elements of a set X, choosing a way
to divide the set E into two subsest with equal sums corresponds to selecting a subset X
of E containing the number 8n(n − 2), and for which
à n
!
ΣE 1 X 1¡ ¢
ΣX = = 8n(n − 2) + ((4n − `) + (4n + `)) = 8n(n − 2) + 8n2 = 8n(n−1).
2 2 `=1
2

In other words, it corresponds to selecting a subset Y of the set E 0 = E \ {8n(n − 2)} for
which ΣY = 8n.
Since max E 0 < 8n < 3 min E 0 , such a set Y contains exactly 2 elements. So the
corresponding sets Y are the sets of the form {4n − `, 4n + `} with 1 6 ` 6 n. There exist
n such sets, so E is n-divided as desired. ¤
JBMO 2021 Geometry. Solutions 16

GEOMETRY. SOLUTIONS

G1. Let ABC be an acute scalene triangle with circumcenter O. Let D be foot of an
A-altitude in triangle ABC and let E denote intersection of lines BC and AO. Let l be
a line through E perpendicular to AO. Let l intersect AB and AC at K, L respectively.
Denote by ω circumcircle of triangle AKL. Line AD intersects ω again at X. Prove that
the circumcircles of triangles ABC, DEX and ω have a common point.
(Proposed by Boris Stanković, Bosnia and Herzegovina)
Solution 1.

Let us denote angles of triangle ABC with α, β, γ in a standard way. We easily get
that ∠BAD = 90◦ − β = ∠OAC and ∠CAD = ∠BAO = 90◦ − γ.
Using the fact that lines AE and AX are isogonal with respect to ∠KAL we can
conclude that X is an A-antipode on ω. (This fact can be purely angle-chased, for
example we have ∠KAX + ∠AXK = ∠KAX + ∠ALK = 90◦ − β + β = 90◦ which implies
∠AKX = 90◦ ) Now let us denote F point on line AE such that XF ⊥ AE. Using that
AX is a diameter of ω and ∠EDX = 90◦ it’s clear that F is the intersection point of ω and
the circumcircle of DEX. Now it suffices to show that ABF C is cyclic. Now we have that
∠KLF = ∠KAF = 90◦ − γ and from ∠F EL = 90◦ we have that ∠EF L = γ = ∠ECL
so quadrilateral EF CL is cyclic. Now we have that ∠AF C = ∠EF C = 180◦ − ∠ELC =
∠ELA = β (where last equality holds because of ∠AEL = 90◦ and ∠EAL = 90◦ − β).¤
JBMO 2021 Geometry. Solutions 17

Solution 2.

As in the first solution we have that ∠BAD = 90◦ − β = ∠OAC and that AX is the
diameter of ω. Also we note that ∠ALK = β, ∠KLC = 180◦ − β = ∠KBC so BKCL
is cyclic. Let AO intersect circumcircle of ABC again at A0 . We will show that A0 is
the desired concurrence point. Obviously AA0 is the diameter of circumcircle of triangle
ABC so ∠A0 CA = 90◦ which implies that A0 CLE is cyclic. From power of point E we
have that EK · EL = EB · EC = EA · EA0 so we can conclude that A0 ∈ ω. Now using
the fact that AX being the diameter of ω implies ∠AXA0 = 90◦ we have that DXA0 E is
cyclic because of ∠EDX = 90◦ which finishes the proof. ¤
JBMO 2021 Geometry. Solutions 18

b = 90◦ . If
G2. Let P be an interior point of the isosceles triangle ABC with A

∠P AB + ∠P BC + ∠P CA = 90◦ ,

prove that AP ⊥ BC.


(Proposed by Mehmet Akif Yıldız, Turkey)
Solution.
A

P
K

x+y y
B C
x D

Let D be the point on BC with AD ⊥ BC. If AP is not perpendicular to BC,


without loss of generality, assume P is inside the triangle ABD. Write ∠P BC = x + y
and ∠P CB = y. From the given angle equality, it is easy to see that ∠P AD = x. Let
F be the point on AD with |P A| = |P F |. Firstly, we have ∠P F D = x. Then, when we
consider the triangle P BC, we have

|F B| = |F C| and ∠P F C − ∠P F B = 2x = 2 · (∠P BC − ∠P CB)

This implies F is the circumcenter of the triangle P BC, and hence we get |F B| = |F P |.
On the other hand, let K be the point on AB such that P K ⊥ AD. Since ∠AKF = 90◦
and KP ⊥ AF , we get |F B| > |F K| > |F P |, which leads to a contradiction. As a result,
we conclude that AP ⊥ BC. ¤

G2B18 . Let P be a point inside the isosceles triangle ABC with ∠BAC = 90◦ . Prove
that the straight lines AP and BC are perpendicular if and only if

∠P AB + ∠P BC + ∠P CA = 90◦ .

Solution 1. 00 ⇒00 Let D be the midpoint of the side BC. Then the straight lines AD
and BC are perpendicular. Suppose that the straight lines AP and BC are perpendicular.
It follows that the point P is a point of the segment AD (see figure).
18
Proposed by PSC.
JBMO 2021 Geometry. Solutions 19

Because AD is the perpendicular bisector of the segment BC we obtain two pairs of


congruent triangles: BDP and CDP , ABP and ACP . If we denote ∠P BC = ∠P CB =
α, then ∠P BA = ∠P CA = 45◦ − α and ∠P AB = ∠P AC = 45◦ . We have

∠P AB + ∠P BC + ∠P CA = 45◦ + α + 45◦ − α = 90◦ .


00
⇐00 Suppose that the point P lie inside of the triangle ABC such, that

∠P AB + ∠P BC + ∠P CA = 90◦ .

Let D be the midpoint of the side BC. It follows that the straight lines AD and BC
are perpendicular. If AP is not perpendicular to BC, without loss of generality, assume
that P is inside of the triangle ABD (see figure).

Let F be the point on AD with |P A| = |P F | and Q is the intersection point of the



segments P C and AD. Denote ∠P AF = ∠P F A = x2 and ∠QBC = ∠QCB = 45 2+y . We

obtain that ∠P AB = 45◦ − x2 , ∠P CA = 45 2−y .. From the given angle equality, it is easy
to see that ∠P BQ = x2 . Because ∠P BQ = ∠P F Q = x2 , it follows that the quadrilateral
BPQF is cyclic and ∠P QB = ∠P F B = 45◦ + y. We have
JBMO 2021 Geometry. Solutions 20

135◦ − y 135◦ − x − y
∠F BP = ∠F P B = , ∠F CP = ∠F P C = .
2 2
From the isosceles triangles BP F and F P C we obtain the equalities |F B| = |F P | =
|F C|. This implies that F is the circumcenter of triangle P BC.
On the other hand, let K be the point on AB such that the line KP and AF are
perpendicular. Since ∠AKF = 90◦ , we get |F B| > |F K| > |F P |, which leads to a
contradiction. As a result, we conclude that the lines AP and BC are perpendicular. ¤

Solution 2. ” ⇒ ” The same proof as in the Solution 1.


00
⇐00 We apply the trigonometrical version of Ceva concurrence theorem: The lines
AP , BP and CP are concurrent in P if and only if
sin ∠P AC sin ∠P BA sin ∠P CB
· · = 1.
sin ∠P AB sin ∠P BC sin ∠P CA
By using the angles notations from Solution 1 we obtain the equality
◦ ◦
sin(45◦ + x2 ) sin( 452 − ( x2 + y2 )) sin( 452 + y2 )
· ◦ · ◦ = 1.
sin(45◦ − x2 ) sin( 452 + ( x2 + y2 )) sin( 452 − y2 )

Let m = tan 452 , n = tan x2 , p = tan y2 . The numbers m, n, p satisfy the relations
0 < m, n, p < 1. By applying trigonometrical calculus we obtain the equalities

sin(45◦ + x2 ) 1+n sin( 452 + y2 ) m+p
◦ x = , 45◦ y = ,
sin(45 − 2 ) 1−n sin( 2 − 2 ) m−p

sin( 452 − ( x2 + y2 )) m − n − p − mnp
45◦ x y = .
sin( 2 + ( 2 + 2 )) m + n + p − mnp
The Ceva relation has the following form:
1 + n m − n − p − mnp m + p
· · = 1.
1 − n m + n + p − mnp m − p
From the last equality we obtain the factorization

2n[m(1 − m) + (1 + m)p2 + (1 + m2 )np] = 0.

Because m(1 − m) + (1 + m)p2 + (1 + m2 )np > 0, then n = tan x2 = 0. So, x = 0. It follows


that the point P lie on the altitude AD and the lines AP and BC are perpendicular. ¤
JBMO 2021 Geometry. Solutions 21

G3. Let ABC be an acute triangle with circumcircle ω and circumcenter O. The perpen-
dicular from A to BC intersects BC and ω at D and E, respectively. Let F be a point
on the segment AD, such that 2 · F D = AE. Let l be the perpendicular to OF through
F . Prove that l, the tangent to ω at E and the line BC are concurrent.
(Proposed by Stefan Lozanovski, North Macedonia)
Solution 1. Let l ∩ BC = G. We will prove that GE is tangent to ω. Let H be the
orthocenter of ABC. It is well-known that HD = DE. From 2F D = AE we get that F
is the midpoint of AH.

Let M be the midpoint of BC. It is well known that M H passes through A0 - the
antipode of A in ω. If S is the second intersection of M H and ω, then ∠HSA ≡ ∠A0 SA =
90◦ , so S lies on the circle with diameter AH, which is centered at F . Therefore, F S = F H
... (1)
Since ∠M SA ≡ ∠A0 SA = 90◦ we have M H ⊥ AS. But since O and F are centers
of ABC and (ASH) and AS is their common chord, we have OF ⊥ AS. Therefore,
M H k OF ... (2)
Since F H and OM are both perpendicular to BC, we get F H k OM . Using (2), we
get that OF HM is a parallelogram. Therefore F H = OM . (This can alternatively be
proven by the well-known fact that AH = 2OM ). Using (1), we get that F S = OM .
Using (2) again, we get that OF SM is an isosceles trapezoid and therefore it’s cyclic.
Using that OF GM is also cyclic (∠OF G = 90◦ = ∠OM G), we get that OF SGM is
cyclic and therefore ∠OSG = ∠OF G = 90◦ . We have HS k OF and OF ⊥ F G, so
HS ⊥ F G. Using (1), we get that F G is the side bisector of SH, so GS = GH. Since
HD = DE, we also get GH = GE. Therefore GS = GH = GE. Finally, we get that
4OSG ∼ = 4OEG (by SSS), so ∠OEG = ∠OSG = 90◦ , i.e. GE is tangent to ω. ¤
JBMO 2021 Geometry. Solutions 22

Solution 2. Let H be the orthocenter of 4ABC and M the midpoint of BC (thus


OM ⊥ BC). Since 2F D = AE, we get that F is the midpoint of AH. It is known that
HD = DE and AH = 2OM . Since F H = 12 AH = OM and F H k OM , we obtain
that F HM O is a parallelogram, hence F O = HM . Since H and E are symmetric with
respect to the line BC, we obtain that EM = HM = F O, which shows that F OM E is an
isoscelles trapezium, hence cyclic. Let G be the intersection point of BC with the tangent
to ω at E. Then∠GEO = 90◦ = ∠GM O, hence OG is the diameter of the circumcircle
of F OM E, and therefore we must have ∠GF O = 90◦ . ¤
JBMO 2021 Geometry. Solutions 23

G4. Let ABCD be a convex quadrilateral with ∠B = ∠D = 90◦ . Let E be the point
of intersection of BC with AD and let M be the midppoint of AE. On the extension of
CD, beyond the point D, we pick a pint Z such that M Z = AE 2
. Let U and V be the
projections of A and E respectively on BZ. The circumcircle of the triangle DU V meets
again AE at the point L. If I is the point of intersection of BZ with AE, prove that the
lines BL and CI intersect on the line AZ.

Solution.

Since M Z = AE 2
= AM = M E then ∠AZE = 90◦ and A, B, E, Z belong to a circle
with center M .
Let O be the projection of M on BZ. Then BO = OZ. Since AU EV is a trapezium,
and M is the midpoint of the diagonal AE, then O is the midpoint of U V . Thus U O =
OV . Since also BU = ZV , then BO = OZ.
Let J be the point of intersection of CD with the circumcircle of the triangle DU V .
Since ∠LDJ = 90◦ , then L and J are antidiametric points of the circle and so the points
L, O, J are collinear with OL = OJ. Since also ∠ZOJ = ∠LOB then the triangles ZOJ
and BOL are equal.
We deduce that ∠OZJ = ∠LBO and therefore ZD is parallel to BL. Since ZD is
perpendicular to AE, it follows that BL is also perpendicular to AE.
Let T be the point of intersection of BL with AZ. Since the triangles ABL and CED
JBMO 2021 Geometry. Solutions 24

are similar, then


BL AL
= (1)
ED CD
Since the triangles ALT and ZDE are similar, then
LT AL
= (2)
ED ZD
From (1) and (2) we deduce that
BL ZD
=
LT CD
from which it follows that CI passes through T .
Alternative Approach: We can also show that BL is perpendicular to AE as follows.
Since ∠ZV E = 90◦ = ∠ZDE, then D, E, Z, V are concyclic. Then ∠IDV = ∠IZE
and ∠IV D = ∠IEZ. So the triangles ZIE and IV D are similar.
Since A, B, E, Z are concyclic, then the triangles ZIE and AIB are similar. Since
D, U, L, V are concylic, then the triangles IV D and IU L are concyclic.
From the above, it follows that the triangles AIB and IU L are similar. Then ∠ILU =
∠ABI, and so A, B, U, L are concyclic. Thus ∠ALB = ∠AU B = 90◦ , i.e. BL is perpen-
dicular to AE. ¤
Comment19 . The initial problem statement is not correct for the case when A lies
between B and E. Furthermore, the addition of point Z and the framing of the problem
in terms of quadrilateral ABCD is artificial and the first step used to get rid of Z is too
simple and omitting it doesnt reduce at all from the complexity of the problem. In the
first solution, it is not proven that O is the circumcenter of circle (DU V ), and proving
this would replicate the same ideas presented in the second solution. The second solution
is based on angle chasing, yet the similarity arguments seem extraneous.

19
All comments are made by PSC.
JBMO 2021 Geometry. Solutions 25

G4B20 . Let ABCD be a convex quadrilateral with ∠ABC = ∠ADC = 90◦ . Let U and
V be the projections of A and C onto BD. Suppose that the perpendicular from D to
AC intersects AC and BC at T and X, respectively. The circumcircle of triangle T U V
intersects AC again at R. If F is the intersection of AC and BD, prove that the lines
XF , BR and AD are concurrent.
Solution.
A B

P R
U
F

V X
T

Since ∠DV C = ∠DT C = 90◦ , DV T C is cyclic. We have


∠DBA = ∠DCA (since ABCD cyclic)
= ∠T V B (since DV T C cyclic)
= ∠T RU (since V T U R cyclic)
and therefore ∠DBA = ∠T RU , which shows that ABU R is cyclic. In particular,
∠ARB = ∠AU B = 90◦ , which shows that BR is perpendicular to AC, hence BR is
parallel to XD.
Let P be the point of intersection of BR with AD. Since triangles ABR and XCT
are similar, we have
BR AR
= . (1)
CT XT
Since triangles ARP and DT C are similar, we have
RP AR
= . (2)
CT DT
From (1) and (2) we deduce that
BR DT RP BR RF
= ⇒ = = .
RP XT XT DT FT
Hence, 4P RF ∼ 4XT F , from which it follows that P , X and F are collinear. ¤
20
Alternative formulation and solution proposed by PSC.
JBMO 2021 Geometry. Solutions 26

G5. Let ABC be an acute scalene triangle with circumcircle ω. A line parallel to BC
cuts the sides AB and AC at points P and Q respectively. Let L (L 6= A) be a point on
ω such that AL k BC. Denote by S the intersection of segments BQ and CP . If K is
the second intersection point of LS with ω, prove that ∠BKP = ∠CKQ.
(Proposed by Ervin Macić, Bosnia and Herzegovina)
Solution 1.

Denote the intersection of SL with P Q as R. We prove BKRP and CKRQ are cyclic
(this is a direct consequence of Reim’s theorem): ∠ALR = ∠ALK = 180◦ ∠ABK =
180∠P BK but also ∠ALR = ∠LRQ = ∠P RK. Now obviously ∠KRQ = 180◦ ∠KCQ.
Now notice that we need to prove ∠BRP = ∠CRQ since ∠BRP = ∠BKP and ∠CKQ =
∠CRQ. This is equivalent to proving BR = CR (∠BRP = ∠RBC and ∠CRQ =
∠RCB). Notice that we would need P R and RM to be the interior and exterior angle
bisectors of ARS. This would mean AR and LR are symmetric w.r.t. bisector of BC,
which is sufficient. From here we can proceed in multiple ways:
Let U and V be the intersections of circles CKRQ and BKRP with CP and BQ
respectively. Since S lies on the radical axis of these two circles, we must have U S · SC =
V S · SB so quadrilateral BU V C is cyclic and thus P U V Q is cyclic (∠U V B = ∠U CB =
∠CP Q). Since P U V Q is cyclic we get ∠QU C = ∠P V B = ∠P RB = ∠CRQ, and we’re
done. ¤
JBMO 2021 Geometry. Solutions 27

Solution 2.

Let M be the midpoint of BC and D the midpoint of AL. The intersection of AS


with P Q is T . We have A, S, M are collinear by Ceva or similar triangles (Thales). We
MT TR
need to prove M, R, D are collinear. This is equivalent to proving M A
= AD = 2T R
AL
but
TR ST MT 2ST
AL
= SA so it suffices to prove M A = SA and here we are basically done since A, T, S, M
lie on one line:
SA 2ST SA 2ST
= ⇔1− =1−
MA MT MA MT
MS M S − ST MT M S − ST ST
⇔ = ⇔ = =1−
MA MT MA MS MS
ST AT
⇔ =
MS MA
AQ PQ
which is true since AC
= BC
. ¤

Solution 3. Same notations as Proof 2: We need to prove that M, R, D are collinear.


By Menelaus it’s enough to prove
AD LR M S AT M S AT MA
· · =1⇔1· · =1⇔ = ,
DL RS M A TS MA TS MS
AQ PQ
which is true since AC
= BC
. ¤
JBMO 2021 Geometry. Solutions 28

Solution 4.

Let R0 be the point on P Q such that BR0 = CR0 . We’ll prove S, R0 , L are collinear.
Let M be the midpoint of BC. By Thales we know A, S, M are collinear. Denote by
T the intersection of AS with P Q. We prove that R0 T is the interior angle bisector of
∠AR0 S by
Lemma. For a given triangle ABC, let P, Q ∈ BC, such that Q ∈ (BC) and B ∈ (P Q).
If PP B
C
= BQ
QC
and ∠P AQ = 90◦ , then AP , AQ are the exterior and, respectively, interior
angle bisectors of ∠BAC.

Proof. Let X and Y be the intersections of the line through Q perpendicular to AQ with
AB and AC respectively. By similar triangles (4BXQ ∼ 4AP B and 4QY C ∼ 4P AC.
We get these since XY is parallel to AP ) we have QX = BQ · PAPB and QY = QC · PAPC .
Now obviously QX QY
= 1, by the condition, hence by SAS congruence (4AXQ and 4AY Q)
the conclusion follows.
Apply this lemma on triangle AR0 S and the segments RT and RM . From here the
AT TS
collinearity is obvious since ∠AR0 P = ∠LR0 Q = ∠P R0 S. (The condition AM = M S
can
be checked to be true by simple similar triangles or Thales). ¤
JBMO 2021 Number Theory. Solutions 29

NUMBER THEORY. SOLUTIONS

NT1. Find all positive integers a, b, c such that ab + 1, bc + 1 and ca + 1 are all equal to
factorials of some positive integers.
(Proposed by Nikola Velov, North Macedonia)
Solution. Because of symmetry, we can assume that a ≥ b ≥ c. In particular, this means
that ab + 1 ≥ ac + 1. Now if ac + 1 = x! and ab + 1 = y!, we have

x! = ac + 1 ≤ ab + 1 = y!

So x! | y! or ac + 1 | ab + 1. From here we obtain:

ac + 1 | (ab + 1) − (ac + 1) = a(b − c).

Using that ac+1 and a are relatively prime, this means that ac+1 | b−c. We conclude
that either b = c or ac + 1 ≤ b − c. However, we also see that 0 ≤ b − c < b ≤ a < ac + 1,
which means that we must have b = c.
Let bc + 1 = z! for some positive integer z. Because b = c, this means that b2 + 1 = z!,
but this is only possible if z = 2 and b = 1. Indeed, if z ≥ 3, then 3 | z!, so 3 | b2 + 1.
However, this would mean that b2 ≡ 2 (mod 3), which is not possible. Checking z = 1
directly we see that this is not possible as well, so b = c = 1.
Finally, a + 1 = ac + 1 = x!, so a = x! − 1 for some positive integer x. Because a
is also a positive integer, we must have x ≥ 2. Taking symmetry into consideration we
obtain that all solutions (a, b, c) are of the form (x! − 1, 1, 1), (1, x! − 1, 1) or (1, 1, x! − 1)
for some positive integer x ≥ 2. We can easily see that all such triples indeed satisfy the
condition of the problem. ¤
JBMO 2021 Number Theory. Solutions 30

NT2. The real numbers x, y and z are such that x2 + y 2 + z 2 = 1.


a) Determine the smallest and the largest possible values of xy + yz − xz.
b) Prove that there does not exist a triple (x, y, z) of rational numbers, which attains
any of the two values in a).
(Proposed by Bulgaria)

Solution. a) We have xy+yz−xz ≥ −(x2 +y 2 +z 2 ) = −1 ⇔ (x+y)2 +(y+z)2 +(x−z)2 ≥ 0


and equality holds only for (± √13 , ∓ √13 , ± √13 ). On the other hand, (x + z − y)2 ≥ 0 ⇔
xy + yz − xz ≤ 21 and equality holds for example when (0, √12 , √12 ).

b) The minimum case is ruled out since 3 is irrational. For the maximum case, it is
enough to consider x2 +y 2 +z 2 = 1 with y = x+z – that is, the equation x2 +z 2 +(x+z)2 =
1. Suppose the latter has a rational solution and write it as x = pr , z = rq , where p, q, r 6= 0
are integers. Then p2 + q 2 + (p + q)2 = r2 ⇔ (2p + q)2 + 3q 2 = 2r2 . Now modulo 3 gives
that 2p + q and r are divisible by 3, whence q (and thus p) is divisible by 3. Writing
p = 3p1 , q = 3q1 , r = 3r1 , we reach (2p1 + q1 )2 + 3q12 = 2r12 , which is the same equation
as the above one for p, q and r. Finally, if the integer s is such that 3s+1 does not
divide r, then performing the above s more times will yield an equation of the form
(2p0 + q 0 )2 + 3q 02 = 2r02 , with the right-hand side not divisible by 3, which is impossible.¤
JBMO 2021 Number Theory. Solutions 31

NT3. For a given set A = {x1 , x2 , x3 , x4 , x5 } of five positive integers denote the sum of
its elements by SA . Let TA denote the number of triples (i, j, k) with 1 6 i < j < k 6 5
for which xi + xj + xk divides SA . Among all sets of five positive integers find, with proof,
the maximum value that TA can attain.
(Proposed by Boris Stanković, Bosnia and Herzegovina)

Solution. We will prove that maximum value TA can attain is 4. Let A = {x1 , x2 , x3 , x4 , x5 }
be set of five distinct positive integers such that x1 < x2 < x3 < x4 < x5 . Call triple
(i, j, k) with 1 6 i < j < k 6 5 good if xi +xj +xk divides SA . Obviously by size argument
triplets (3,4,5), (2,4,5),(1,4,5),(2,3,5), (1,3,5) aren’t good because for example
x5 + x3 + x1 | SA ⇐⇒ x5 + x3 + x1 | x2 + x4
which is impossible since x5 > x4 and x3 > x2 . Analogously we can show that any triple
of form (x, y, 5) where y > 2 isn’t good.
Because of that number of good triplets is at most 5 and only triplets (1,2,5), (2,3,4),
(1,3,4), (1,2,4), (1,2,3) can be good. But if triplets (1,2,5) and (2,3,4) are simultaneously
good we have that:
x 1 + x2 + x5 | x3 + x4 ⇒ x5 < x 3 + x4 (1)
and
(1)
x2 + x3 + x4 | x1 + x5 ⇒ x2 + x3 + x4 6 x1 + x5 < x 1 + x3 + x4 < x 2 + x3 + x4 ,
which is impossible. Therefore, TA 6 4.
To show that TA = 4 is possible consider numbers 1, 2, 3, 4, 494. This works because
6 | 498, 7 | 497, 8 | 496, and 9 | 495. ¤

Remark. Motivation for construction is to realize that if we choose x1 , x2 , x3 , x4 we can


get all the conditions x5 must satisfy. Let S = x1 + x2 + x3 + x4 . Now only restrictions
for our choice of x5 is that the following must be true: S − xi | xi + x5 ←− x5 ≡ −xi
mod (S − xi )∀i ∈ {1, 2, 3, 4}. If someone is familiar with the Chinese Remainder Theorem
it is obvious that if all numbers S − x1 , S − x2 , S − x3 , S − x4 are pairwise coprime, such x5
must exist. To make all these numbers pairwise coprime it’s natural to take x1 , x2 , x3 , x4
to be all odd and then solve mod 3 issues. Fortunately it can be seen that 1, 5, 7, 11 easily
works because 13, 17, 19, 23 are pairwise coprime.
However, even without the knowledge of this theorem it makes sense intuitively that
this system must have a solution for some x1 , x2 , x3 , x4 . By taking (x1 , x2 , x3 , x4 ) =
(1, 2, 3, 4) we get pretty simple system which can be solved by hand rather easily.
There are many other possible constructions and they don’t really require knowledge
of CRT but they all require some kind of insight which often very important for senior
number theory problems. Therefore we can consider this problem a great transition from
junior to senior olympiad problems because it has some bounding ideas which are pretty
standard for junior problems while finding example requires more senior approach.
JBMO 2021 Number Theory. Solutions 32

NT4. Dragoş, the early ruler of Moldavia, and Maria the Oracle play the following game.
Firstly, Maria chooses a set S of prime numbers. Then Dragoş gives an infinite sequence
x1 , x2 , . . . of distinct positive integers. Then Maria picks a positive integer M and a prime
number p from her set S. Finally, Dragoş picks a positive integer N and the game ends.
Dragoş wins if and only if for all integers n ≥ N the number xn is divisible by pM ;
otherwise, Maria wins. Who has a winning strategy if the set S must be: a) finite b)
infinite?
(Proposed by Bulgaria)
Solution. We show that in both cases Dragoş can win.
Suppose firstly that Maria chooses the finite set S = {p1 , p2 , . . . , pk }, where p1 < p2 <
. . . < pk . Then Dragoş can use the sequence xn = (p1 p2 · · · pk )n (which is increasing and
hence consists of distinct terms). Now, no matter what M and p Maria picks, Dragoş can
give N = M in order to win – indeed, xn is divisible by pn for each p in S and hence by
pM for all n ≥ N .
Now consider the case when Maria chooses the infinite set S = {p1 , p2 , . . .}, where
p1 < p2 < . . .. Then Dragoş can use the sequence xn = (p1 p2 · · · pn )n (which is increasing
and hence consists of distinct terms). Now, no matter what M and pk Maria picks, Dragoş
can give N = max(M, k) in order to win – indeed, xk is divisible by pnk for each pk in S
whenever n ≥ k and hence by pM for all n ≥ max(M, k). ¤
JBMO 2021 Number Theory. Solutions 33

NT5. Find all pairs of integers (x, y) such that x2 + 5y 2 = 2021y.


(Proposed by Bulgaria)
Answer. (0, 0), (±390, 100), (±408, 289)

Solution. The cases x = 0 and y = 0 are immediate; we can without loss of generality
treat x > 0 and hence y > 0. Clearly, x2 = y(2021 − 5y) and the greatest common
divisor of the multipliers on the right divides 2021 = 43 · 47. Since 2021 − 5y > 0, i.e.
y ≤ 404, we have the following possibilities: 43 | y, 47 | y and GCD(y, 2021 − 5y) = 1.
If y = 43z, then x = 43t and we get t2 = z(47 − 5z), z ≤ 9 and a direct verification
shows there is no solution. If y = 47z, then x = 47z and we get t2 = z(43 − 5z), z ≤ 8
and a direct verification shows there is no solution. In the last case we necessarily have
y = m2 and 2021 − 5y = n2 for positive integers m and n, i.e. n2 + 5m2 = 2021. Now
we get 5m2 < 2021, i.e. m ≤ 20; moreover m is not divisible by 3 (else n2 ≡ 2 (mod 3))
and does not give remainder 0, 2 or 5 when divided by 7 (else n2 ≡ 5, 6 (mod 7)).
The remaining ones are m = 1, 4, 8, 10, 11, 13, 17, 20 and for 2021 − 5m2 we calculate
442 < 1941 < 2016 < 452 (so m = 1 and m = 4 do not work), 412 < 1701 < 422 (so
m = 8 does not work), 1521 = 392 (respectively y = 100), 372 < 1376 < 1416 < 382 (so
m = 11 and m = 13 do not work), 576 = 242 (respectively y = 289) and 42 < 21 < 52 (so
m = 20 does not work). ¤
JBMO 2021 Number Theory. Solutions 34

NT6. Given a positive integer n > 2, we define f (n) to be the sum of all remainders
obtained by dividing n by all positive integers less than n. For example dividing 5 with
1, 2, 3 and 4 we have remainders equal to 0, 1, 2 and 1 respectively. Therefore f (5) =
0 + 1 + 2 + 1 = 4.
Find all positive integers n > 3 such that f (n) = f (n − 1) + (n − 2).
(Proposed by Cyprus)

Solution. Given any d < n we write ad for the remainder when n − 1 is divided by d and
bd for the remainder when n is divided by d.
If d|n then we have ad = d − 1 and bd = 0. If d - n then we have bd − ab = 1. Thus

X X
f (n) − f (n − 1) = (bd − ad ) + (bd − ad )
d-n d|n
X X
= 1− (d − 1)
d-n d|n
X X
= 1− d
d d|n

= (n − 1) − [σ(n) − n]
= 2n − 1 − σ(n)

Here, all sums are over all integers d ∈ {1, 2, . . . , n − 1} satisfying the claimed properties,
σ(n) is the sum of all positive divisors of n (including n) and d(n) is the total number of
positive divisors of n (including n).
So f (n) = f (n − 1) + (n − 2) if and only if σ(n) = n + 1. But since 1, n are divisors
of n, then σ(n) > n + 1 with equality if and only if n is a prime number.

Note: Several other modifications


√ are possible. E.g. one can ask to find all n for which

f (n) > f (n − 1) + (n − n − 2). This results to finding all n for which σ(n) < n + 1 + n
which again gives that n must
√ be a prime number. (Otherwise it has 1, n and another
number greater or equal to n as divisors.)
Another modification is to ask to find all n such that f (n) = f (n − 1) − 1. This
happens exactly when n is a perfect number. ¤
JBMO 2021 Number Theory. Solutions 35

NT7. Alice chooses a prime number p > 2 and then Bob chooses a positive integer
n0 . Alice, in the first move, chooses an integer n1 > n0 and calculates the expression
s1 = nn0 1 + nn1 0 ; then Bob, in the second move, chooses an integer n2 > n1 and calculates
the expression s2 = nn1 2 + nn2 1 ; etc. one by one. (Each player knows the numbers chosen
by the other in the previous moves.) The winner is the one who first chooses the number
nk such that p divides sk (s1 + 2s2 + · · · + ksk ). Who has a winning strategy?
(Proposed by Borche Joshevski, North Macedonia)

Solution. We will prove that for any prime p > 2, Bob can win by choosing n0 = (p−1)2 .
Then (
1 if p | n1
s1 = nn0 1 + nn1 0 ≡ 6≡ 0 (mod p) for any n1 > n0 .
2 if p - n1
We will use the following two properties:
1) If p | nk then it is obvious that for nk+1 = pnk > nk we have p|sk+1 .
2) If p - nk and nk is odd then for nk+1 = (p − 1)(pnk + 1) > nk we have
n
nk+1 ≡ −1 (mod p), and sk+1 = nk k+1 + nnk+1
k
≡ 1 + (−1)nk ≡ 0 (mod p).

From 1) and 2) it follows that: If one of the players chooses nk to be odd or even
number divisible by p but does not win in his current move then the other can win in the
next move. If the player cannot chose nk for which he wins, then it is clear that the only
possibility not to lose is to choose nk to be an even and p - nk .
Let n2j = (p − 1)2m2j for j = 1, , p−12
, where m2j is chosen so that n2j > n2j−1 .
Then (
nj n 1 if p - nj−1 nj
sj = nj−1 + nj j−1 = 6≡ 0 (mod p)
2 if p - nj−1 nj
and for k < p − 1 we have s1 + 2s2 + + ksk ≡
( (
2(1 + 2 + + k − 1) + k if p | nk k2 if p | nk
≡ = 6≡ 0 (mod p)
2(1 + 2 + + k) if p - nk k(k + 1) if p - nk

and s1 + 2s2 + + (p − 1)sp−1 ≡ 2(1 + 2 + ... + (p − 1)) ≡ 0 (mod p).


As p − 1 is even, we get that the winning move belongs to Bob. ¤
JBMO 2021 Combinatorics. Solutions 36

COMBINATORICS. SOLUTIONS

C1+NT. In Mathcity, there are infinitely many buses and infinitely many stations. The
stations are indexed by the powers of 2: 1, 2, 4, 8, 16, . . . Each bus stops by finitely many
(distinct) stations, and the bus number is the sum of indices of the stations it stops by.
The bus numbers form an arithmetic progression with common difference r and whose
first term is the favourite number of the mayor.
For which positive integers r is it certain that, no matter the favourite number of the
mayor, there is a bus stopping by any given m stations (and possibly, other stations)?
(Proposed by Savinien Kreczman and Martin Rakovsky, France)

Solution. If r is even and the favourite number of the mayor is 2, no bus will ever go by
the station number 1. Thus, even numbers r do not satisfy the problem requirement.
If r is odd, consider m bus stations with numbers 2a1 , . . . , 2am , such that a1 < a2 < . . . <
am , and let f be the favourite number of the mayor. Since r is odd, it is coprime with
2am +1 , and thus there exists a positive integer q such that rq ≡ 1 (mod 2am +1 ). Then,
let s be a positive integer such that s ≡ −1 − f (mod 2am +1 ). According to the problem
statement, there is a bus with number f + r(qs).
Since f + r(qs) ≡ f + s ≡ −1 ≡ 20 + 21 + . . . + 2am (mod 2am +1 ), this bus will go by
each of the stations 20 , 21 , . . . , 2am . In particular, it goes by each of our initial m stations,
and thus r satisfies the problem statement.
In conclusion, the solutions of the problem are the odd integers r. ¤
JBMO 2021 Combinatorics. Solutions 37

C2. Let n be a positive integer. We are given a 3n × 3n board whose unit squares are
colored in black and white in such way that starting with the top left square, every third
diagonal is colored in black and the rest of the board is in white. In one move, one can
take a 2 × 2 square and change the color of all its squares in such way that white squares
become orange, orange ones become black and black ones become white. Find all n for
which, using a finite number of moves, we can make all the squares which were initially
black white, and all squares which were initially white black.

(Proposed by Boris Stankovć and Marko Dimitrić, Bosnia and Herzegovina)

Solution 1. (by proposers) Firstly, observe that if we change the color of one square 3
times, it goes back to its original color. As the final configuration does not depend on the
order of moves and placing a move on a square 3 times does not change the configuration,
we can suppose that a move is placed on any square exactly 0, 1 or 2 times. This further
implies that a white square must change its colour 2 (modulo 3) times and that a black
square must be changed 1 (modulo 3) times.
Suppose we can reverse the colours of the squares, as required in the problem state-
ment, for some 3n × 3n board.
Claim: n cannot be odd.
Proof. Let us associate each 2×2 square with its top left unit square. Let’s take a look
at the first column. Its top square is white and is included in only one 2 × 2 square, so
that square has to be placed 2 times. The second square in this column is also white and
is included in its 2 × 2 square as well as in the one associated with the first unit square in
the column. This square has already been turned black by taking the 2 × 2 square of the
first unit square in this column twice, so its own 2 × 2 square has to be taken 0 times.
We use similar arguments to show that next four 2 × 2 squares have to be called 1,1,1 and
0 times. After that we have the white square included only in its own 2 × 2 square and
in the one above it, which has been taken 0 times, so we have the same situation we had
in the beginning. That means that in the first column numbers of times successive 2 × 2
squares have to be taken make a cycle (2,0,1,1,1,0). Now take a look at the last square
in this column. It is obviously black. Also, its 2 × 2 square is not included in the board,
so this unit square is included only in the 2 × 2 square of a unit square above it, which
has been taken 0 times if 6 | 3n + 3 and 1 time if 6 | 3n. We know that this square must
change its color once and that is only possible if 6 | 3n, which means that 2 | n. Here is
the construction for n = 2:
JBMO 2021 Combinatorics. Solutions 38

Here the number inside the unit square denotes the number of times the 2 × 2 square
with that unit square as its top left square is taken. Now for n = 2k, k > 1, we can
partition the board into several disjoint 6 × 6 boards, which we can solve separately,
which will solve the entire board (it is trivial that those 6 × 6 boards are colored the same
way the board in case n = 2 is colored). ¤

Solution 2. (by Milica Vugdelić, Serbia) As in the first solution, we can assume that a
move is placed on every square 0, 1 or 2 times. Also, use the same fact that white squares
must be change colour 2 (modulo 3) times and that black squares must change colour 1
(modulo 3) times. Use the same construction for even n. Suppose that we can obtain the
required board for some n.
Claim 1: The number of moves is divisible by 3.
Proof. Fix a sequence of moves and assign to each unit square the number of times
it has changed colour in the sequence. Then the sum of all assigned values is equal to
2W + B modulo 3, where B and W are the number of black and white squares on the
initial board, respectively. It is easy to see that 3 | W and 3 | B so 3 | 2W + B. On
the other hand, the sum of all assigned values is just 4 times the number of moves (every
change of colours of a 2 ×2 square adds 4 to our sum). So the number of moves is divisible
by three. ¤

Claim 2: n cannot be odd.


Proof. Consider the set of every other square in every other row of the board (if we set
the coordinate sysem with the top left square as (1, 1) and the bottom right as (3n, 3n),
we see that these squares are just the ones with both coordinates even).Observe that every
move changes the colour of exactly one of these squares, so the sum of assigned values of
these squares must be equal to the number of moves. If 2 - n (n = 2k + 1, k > 0) then we
3n 2
have = (3k +1)2 squares with both coordinates even, of which b = 23 (3k)2 +3 = 6k +3
2
are initially black and the rest are white. But the sum of assigned values of these squares
will be 2((3k + 1)2 − b) + b modulo 3 and since 3 | b we see that this is not divisible by
three. But then the number of moves is not divisible by three, a contradiction. ¤
JBMO 2021 Combinatorics. Solutions 39

Solution 3. (by Mateja Vukelić, Serbia) As in the first two solutions, we can assume
that a move is placed on every square 0, 1 or 2 times. Also, use the same fact that white
squares must be change color 2 (modulo 3) times and that black squares must change
color 1 (modulo 3) times. Use the same construction for even n. Let power of each
square be 0 if it is white, 1 if it is orange and 2 if it is black. Now color the board as a
chessboard in blue and red, and let the top left square be red. Define the P owerOf Red
as the sum of powers of all red squares.We define the P owerOf Blue the same way. Let
S = P owerOf Red − P owerOf Blue.
Claim 1: After each move, S stays the same modulo 3.
Proof. It is obvious that each move affects exactly 2 red squares and 2 blue ones.
Notice that, when looking at powers of squares modulo 3, in each move we add 1 to the
power of each square. That means that each move increases both the P owerOf Blue and
P owerOf Red by 2 each, so S increases by 2 − 2 = 0, which means it stays the same. ¤

Claim 2: n cannot be odd.


Proof. Let n = 2k + 1. The number of red squares is R = (3k + 1)(6k + 3) + 3k + 2
and the number of blue squares is B = (3k + 1)(6k + 3) + 3k + 1. Notice that, at the
beginning if some square blue square is black, its entire diagonal is black and it contains
3m squares, so the number of all black blue squares is divisible by 3, which means that the
P owerOf Blue is divisible by 3. We use the same argument to show that, at the beginning,
the P owerOf Red is divisible by 3 as well. This means that the S is divisible by 3. Now
let’s take a look at our desired endboard. Now the number of black blue squares is the same
as the number of white blue squares on the starting board, which is B minus some number
divisible by 3. This equals 3k + 1 modulo 3, so now P owerOf Blue ≡ 2 ∗ (3k + 1)(mod
3). We use the same argument to show that now P owerOf Red ≡ 2 ∗ (3k + 2)(mod 3), so
S ≡ 2 ∗ (3k + 2 − 3k − 1) ≡ 2(mod 3). However, we already proved that after each move
S stays the same modulo 3, so there is no way to apply some sequence of moves which
will turn our starting board into desired endboard. ¤

Remark. The problem can be solved for every n × n board, not just 3n × 3n, but
for choices of n not divisible by 3 the board looks rather ugly, so we chose to focus on
this subproblem. However, if one wishes to solve the general problem, our period/cycle
argument can quickly show that for every positive integer k, n = 6k + 1,n = 6k + 4 and
n = 6k + 5 doesnot work. This leaves us with n = 6k + 2.
JBMO 2021 Combinatorics. Solutions 40

Here we have 8 × 8 board. Again, the number inside the unit square denotes the number
of times the 2 × 2 square with that unit square as its top left square is taken. We can
get that this must be part of correct strategy if one exists simply by starting from the
top left square, seeing how many times its square must be changed so that it turns black,
and going down the column and repeating the process for the second column. However
we now get that the bottom square in the second column has changed its color 2 times,
but since the board is (6k + 2) × (6k + 2), we know that this square is black, so it has to
change its color 1 time. This is enough to say that 8x8 board does not work. For other
(6k + 2) × (6k + 2) boards we get the completely same situation because numbers in each
column form a cycle with the length 6 Thus, we proved that even in general case, only
boards which work are 6k × 6k ones. ¤
JBMO 2021 Combinatorics. Solutions 41

C3. We have a set of 343 closed jars, each containing blue, yellow and red marbles with
the number of marbles from each color being at least 1 and at most 7. No two jars have
exactly the same contents. Initially all jars are with the caps up. To flip a jar will mean to
change its position from cap-up to cap-down or vice versa. It is allowed to choose a triple
of positive integers (b, y, r) with b, y, r ∈ {1, 2, ..., 7}, and flip all the jars whose number
of blue, yellow and red marbles differ by not more than 1 from b, y, r, respectively. After
n moves all the jars turned out to be with the caps down. Find all possible values of n.
(Proposed by Bulgaria)

Solution. Call a jar important if each of the quantities of blue, yellow and red marbles
in it is 1, 4 or 7. There are 3 · 3 · 3 = 27 important jars. It is easy to check that any
move flips exactly one important jar. Thus, after an even number of moves, the number
of flipped jars is even. Therefore, the required outcome can appear only after an odd
number of moves which is no less than 27. In order to achieve any such number, make a
move for each of the 27 important jars, followed by any number of parasite pairs of moves
with (say) (b, y, r) = (1, 1, 1). Thus n can be any odd number which is at least 27. ¤
JBMO 2021 Combinatorics. Solutions 42

C4. Alice and Bob play a game together as a team on a 100 × 100 board with all unit
squares initially white. Alice sets up the game by coloring exactly k of the unit squares
red at the beginning. After that, a legal move for Bob is to choose a row or column
with at least 10 red squares and color all of the remaining squares in it red. What is the
smallest k such that Alice can set up a game in such a way that Bob can color the entire
board red after finitely many moves.
(Proposed by Nikola Velov, North Macedonia)

Solution. We will show by induction on m + n the following:


Fix integers m, n ≥ 1 and 0 ≤ a < n, 0 ≤ b < m. Assume that the game is being
played on a board with m rows and n columns, but assume that a row is a legal move for
Bob if it has at least a red squares and a column is a legal move for Bob if it has at least
b red squares. Then if it is possible to win the game, there must be at least ab squares
initially red (colored by Alice at the beginning of the game).
The base case m = n = 1 and a = b = 0 is trivial.
Assume the contrary. This means that there are strictly less than ab red squares
initially. If a = 0 or b = 0 this is trivially not possible. Otherwise, provided the game
can be won by the children, there is either a row with at least a red squares or a column
with at least b red squares (if this is not the case then there are no legal moves). Assume
without loss of generality that Bob’s first turn is a row. Now make a new board by cutting
out this row (which is now completely red).
We obtain a new board with a1 = a and b1 = b − 1, because all columns from the first
board now have at least one red square (from the row we cut out). The total number of
red squares on the new board is less than ab − a = a(b − 1) = a1 b1 , while the board is
(m − 1) × n. We also have 0 ≤ a1 < n and 0 ≤ b1 < m − 1. But this board cannot be
won by induction (we have (m − 1) + n < m + n), so the original board cannot be won as
well. We conclude that we must have at least ab squares which are initially red.
Now taking n = m = 100 and a = b = 10 we see that we need at least 10 × 10 red
squares, or in other words, k ≥ 100. To show that this is indeed possible, notice that
Alice can color the upper left corner 10 × 10 subsquare red. Now Bob first makes moves
on the top 10 rows, then on all the columns, in this order. This makes the entire board
red, so k = 100. ¤
JBMO 2021 Combinatorics. Solutions 43

C5. Let A be a subset of {1, 2, 3, . . . , 2021} such that whenever a, b, c are three not
necessarily distinct elements of A, then |a + b − c| > 10.
What is the largest possible size of A?
(Proposed by Cyprus)

Solution. The set A = {1016, 1017, . . . , 2021} has the necessary property as a, b, c ∈ A
implies that a + b − c > 1016 + 1016 − 2021 = 11. Notice that this set has 1006 elements.
We will show that this is optimal.
Let k be the minimal element of A. Then k = |k + k − k| > 10. For every m, at least
one of m, m + k − 10 does not belong to A, since k + m − (m + k − 10) = 10.
Claim 1: A contains at most k − 11 out of any 2k − 22 consecutive integers.
Proof: We can partition the set {m + 1, m + 2, . . . , m + 2k − 22} into k − 11 pairs as
follows:

{m + 1, m + k − 10}, {m + 2, m + k − 9}, . . . , {m + k − 11, m + k − 22},

It remains to note that A can contain at most one element of each pair. ¤
Claim 2: A contains at most (t + k − 11)/2 out of any t consecutive integers.
Proof: Write t = q(2k − 22) + r with r ∈ {0, 1, 2 . . . , 2k − 21}. From the set of the first
q(2k − 22) integers, by Claim 1 at most q(k − 11) = t−r 2
can belong to A. From the last
r integers, at most min{r, k − 11} can belong to A. Theorefore, at most

t−r t − r r + k − 11 t + k − 11
+ min{r, k − 11} ≤ + =
2 2 2 2
of the t consecutive integers can belong to A, as claimed. ¤
By Claim 2, amongst k + 1, k + 2, . . . , 2021 at most

(2021 − k) + (k − 11)
= 1005 integers belong to A.
2
Since amongst {1, 2, . . . , k} only k belongs to A, then A has at most 1006 elements as
required. ¤
JBMO 2021 Combinatorics. Solutions 44

C6. Given an m × n table consisting of mn unit cells. Alice and Bob play the following
game: Alice goes first and the one who makes move colors one of the empty cells with one
of the given three colors. Alice wins if there exist 3 cells placed diagonally (as in one of
the figures below), having three different colors. Otherwise Bob is the winner. Determine
the winner for all cases of m and n where m, n ≥ 3.

(Proposed by Toghrul Abbasov, Azerbaijan)

Solution. For the sake of simplicity, we will label colors as a, b, c. Assume that m ≥
5, n ≥ 4 or m ≥ 4, n ≥ 5. Without loss of generality, consider second case (for other case,
we may rotate table). Since m ≥ 4, n ≥ 5, we can take the following subfigure from the
table:
1 5
2 6
3
7 4

Assume Alice colors 3 as a. After Bob makes move, at least one of triples {1, 2, 4} and
{5, 6, 7} will remain uncolored. Suppose it is {1, 2, 4}. Then, Alice colors 2 as b. After
next move of Bob, again at least one of 1 and 4 is still empty. Alice will choose uncolored
one and color it with c. Thus, Alice wins.
Now, consider the case m = n = 4: Let’s split the squares of the 4 × 4 table in pairs
as follows:
1 8 3 5
7 1 5 4
3 6 2 8
5 4 6 2

Whenever Alice chooses a cell, Bob will color its pair using same color as Alice. Thus,
Bob wins (since for any diagonal of 3 squares, 2 of the squares will be paired).
Now, suppose m = 3, n = 6 (the symmetric case m = 6, n = 3 is treated similarly).
Similar to above, we split the cells of the table into pairs as follows:

1 9 2 7 3 6
5 1 8 4 6 5
2 8 3 9 4 7

Whenever Alice chooses cell, Bob will color its pair using same color as Alice. Thus,
Bob wins.
Likewise, for m = 3, n = 4 (and n = 3, m = 4) the pairing
JBMO 2021 Combinatorics. Solutions 45

1 5 2 4
6 1 4 3
2 6 3 5

shows that Bob has winning strategy.


Consider now the case m = n = 3.
1 2 3
4 5 6
7 8 9

Look at the following pairs: (4,2) (8,6). Whenever Alice chooses a cell numbered 2, 4,
6, or 8, Bob will color its pair with same color as Alice. If Alice colors 1 (3, 7, and 9 cases
are just rotation) with color a, Bob will color 5 with the color a. Then Bob will color 3
or 7 (which one is empty) with color a after Alices move. If Alice colors 5 with color a,
then Bob will color at least one cell from pairs (1,9) and (3,7) with color a in his next 2
moves. So Bob wins.
Suppose m = 3, n ≥ 7 (the case m ≥ 7, n = 3 is similar). Consider the 3 × 7 subtable
from the top left.
1 2 3 4 5 6 7 ... ...
8 9 10 11 12 13 14 ... ...
15 16 17 18 19 20 21 ... ...

Let Alice color 1 as a.


1. Assume Bob doesnt select 9,17,15,3,11,5,19 in his next move. Then, Alice colors 9
as b. Bob must color 17 as one of a, b (suppose it is a). Then, Alice chooses 11 and
color it as b. Again, Bob must color 5. Finally, Alice will choose 3 and color it as a.
Now, Bob need to make move on both 15 and 19 simultaneously. Alice wins.
2. Assume Bobs next move is either 11 or 5. At least one color is unused. Then, Alice
colors 17 with any color among remaining ones. Now, Bob should make move on 9
and uncolored cell between 5 and 11. Thus, Alice wins.
3. Assume Bobs next move is 3. At least one color is unused. Then, Alice colors 9
with any color among remaining ones. Now, Bob should make move on 15 and 17.
Thus, Alice wins.
4. Assume Bobs next move is 15. At least one color is unused. Then, Alice colors 9
with any color among remaining ones. Now, Bob should make move on 3 and 17.
Thus, Alice wins.
5. Assume Bobs next move is 19. Alice colors 9 as b. Then, Bob must color 17 as one
of a, b (suppose it is a). Then, Alice colors 11 as b. Again, Bob must choose 5 and
color it as one of a, b. Now, there is at least one unused color among 5 and 19. Alice
colors 13 with any color among remaining ones. Bob should make move on 7 and
21. Alice wins.
JBMO 2021 Combinatorics. Solutions 46

6. Assume Bobs next move is 17. Obviously, Bob should color it as a. Otherwise, Alice
may choose remaining color for 9 and win. So, suppose A colors 11 as b. Then, Bob
must color 5 as one of a, b (suppose it is a). Then, Alice colors 13 as b. Bobs next
move is 21. Alice colors 19 as any of unused color among 11 and 13 (this clearly
exists). Now, Bob should make move on 3 and 7. Alice wins.
7. Assume Bobs next move is 9. Obviously he should color it as a. Alice colors 15 as b.
Then, Bob must colors 3 as one of a, b (suppose it is a). Alice colors 11 as b. Again,
Bob must color 19 as one of a, b. Alice colors 13 as different colors from 19 so that
Bob makes his next move on 7. Since there is at least one unused color among 11
and 13, Alice may choose any of remaining ones, and force Bob to color 17 and 21
simultaneously. Alice wins.
Finally, consider the case m = 3, n = 5 (the case n = 3, m = 5 is similar).
1 2 3 4 5
6 7 8 9 10
11 12 13 14 15
Alice will color one of the even-numbered cells in the first move. Since there are 7 (i.e.
an odd number of) even-numbered cells in total, Alice can force Bob to start coloring
odd-numbered cells.
1. If Bob colors cell 1 with a, Alice will color cell 7 with b. Then, Bob will have to
color cell 13 with a or b. Next, Alice will color cell 9 with c. So, Bob will need
to color cell 5, and Alice will color 3 with a. Bobs subsequent move will guarantee
to have at least 11 or 15 to be uncolored. If 11 is remained uncolored, then, Alice
will color it with c, and in the end, 3, 7, 11 will have different colors. Alice wins.
Otherwise, if 15 remained uncolored, Alice will color it with b, and 3, 9, 15 will have
different colors. Alice wins.
2. If Bob colors 5,11 or 15, the case is similar (by symmetry) to 1).
3. If Bob colors 7 with a, Alice will color 1 with b, and then the process will continue
as in case 1). Alice wins.
4. If Bob colors 9, the case is similar (by symmetry) to 3). Alice wins.
5. Assume Bob colors cell 3 with a. Alice will then color 11 with b. Since Bob will
color 7 either a or b, Alice will color 1 with c. Therefore, Bob will choose one of the
colors of 1 or 7 to paint 13. As there will be at most two different colors used for 3
and 13, Alice paints 9 with one of the remaining colors. Since Bob will paint one of
the cells 5 or 15 in the next move, Alice will paint with different color and win.
6. If Bob colors 13, the case is similar (by symmetry) to 5). Alice wins.
Remark. The above solution deals with all cases of m, n ≥ 3. However, it is suggested
to ask for mn ≥ 16, m, n ≥ 3 to make problem easier since the case 3 × 5 is not easy as
other cases.

You might also like